What do you know about the slope of two parallel lines? *
They have the opposite sign of each other
They are reciprocals of each other
They have the same slope
They have negative reciprocal slopes

Answers

Answer 1

Answer:

same slope

Step-by-step explanation:

if it has the same slope then the you will know the two lines are parallel

hope this helps

brainliest?


Related Questions

Salena needs to purchase 9 skirts for a new job. her friend, Jamie goes with her each of the skirts costs $41. Salena says that she can find the total cost by multiplying 9 and 4 and adding that to product of 9 and 1. Jamie disagrees and says that she needs to find the product of 9 and 40 and add that to the product of 9 and 1. Who is correct and why?

Answers

Answer:

Jamie is correct. This is because 41 is a combination of 40 and 1. Salena thinks that the 4 is just a 4, however, the 4 is in the tens place, so the 41 without the 1 is 40. The answer to this 41 times 9 is 369. Not 45.

Step-by-step explanation:

Each skirt is $41

Salena needs 9

(9*40) + (9*1)

360   +   9

    $369

BI
Challenge 2
A: Can you find the
area of the circle?
r=x +1
B: Can you find the
circumference of
the circle?

Answers

Answer:

Area of the circle = pi * r^2 = pi * (x+1) ^ 2

Circumference of the circle = 2* pi * r = 2*pi*(x+1)

Pls I need help ASAP

Answers

Answer:

the answer is 1/8

thank you

if you loved my answer then please like it and mark as brainliest

Tell whether the sum is positive, negative, or zero without adding them.
-8 + 20

Answers

Answer:

The sum is positive.

Step-by-step explanation:

The 20 is a far greater value than -8, so no matter what, when you add them the sum will be positive. Though this would not be the same were the -8 greater than -20.

Hope this helps!

Cliff needs 6 meters of wood for a building project. He already has 3
meters of wood.
Approximately how many feet of wood does Cliff need? (One meter
is approximately 3.3 feet.)
1 foot
3 feet
6 feet
10 feet

Answers

letter B 3 feet

Step-by-step explanation:

he needs three more to get 6 meters of wood

10, because he needs 3 more meters and 3 meters = 9.9 feet

diride 660cm in the ratio of 5:6​

Answers

Answer:

Thus, the measure 660 cm is divided into 300 + 360 in the ratio of 5:6

Step-by-step explanation:

Equations

We are required to divide 660 cm in the ratio 5:6, or 5/6.

Let's call:

x = first part of 660

660 - x = second part of 660

The ratio between them is 5/6.

[tex]\displaystyle \frac{x}{660-x}=\frac{5}{6}[/tex]

The above equation will result in x as the smaller part of 660.

Multiply by (660-x)*6 to eliminate denominators:

[tex]\displaystyle (660-x)*6\cdot\frac{x}{660-x}=(660-x)*6\cdot\frac{5}{6}[/tex]

Operating:

[tex]6x=(660-x)*5[/tex]

[tex]6x=3300-5x[/tex]

Simplifying:

[tex]6x+5x=3300[/tex]

[tex]11x=3300[/tex]

Solving:

[tex]\displaystyle x=\frac{3300}{11}=300[/tex]

The other part is 660 - 300 = 360

Thus, the measure 660 cm is divided into 300 + 360 in the ratio of 5:6

ax+6=15 a is a negative, what must be true about x?

Answers

Answer:

x= 9/a

Step-by-step explanation:

Solve for x for the following pls!
1) 7+3x+1=4x+5
2) 8+2x+18=x+19
3) 6+2x+13=x+26
would appreciate it sm

Answers

Answer:

1.x=3

2. x= -7    

3. x=7

Step-by-step explanation:

guys what is 265 divided by 53

Answers

Answer:

5

Step-by-step explanation:

265/53= 5

Answer:

265÷53=5

Step-by-step explanation:

just get a calculator

I don't understand with this question, can you help me with this pls?

Answers

Answer:

By moving the decimal at the end of the number to the correct amount of places for x 10 you move it once to the right for x 100 you move it right twice.

Step-by-step explanation:

subtract 8 from the input to get the output
divided the input by 3 to get the output

Answers

Answer: that will be 5

Step-by-step explanation:

Answer:

6

Step-by-step explanation:

Which of the following numbers is irrational?

Answers

The second one I think

What’s the difference between m=change in y and change in x and m =y2-y1and x2-x1

Answers

it's the same thing

[tex]slope = \frac{Δy}{Δx} or \frac{change \: in \: y}{change \: in \: x} = \frac{y2 - y1}{x2 - x1} = \frac{rise}{run} [/tex]

(-8) X 2/3 i NEED the answer help me

Answers

-5.33333333333

I used a calulator can I get brainliest please

x+12x-13 = 0 and azo, what
is the value x+lo?​

Answers

Wat this doesn’t make sense

6. The store paid $2.50 for a book and sold it for $5.25. What is the profit as a percent?

Answers

7.75 I hope this helps

The angle measures of triangle QRS are given:

The measure of angle Q is (5x−15)°.
The measure of angle R is 75°.
The measure of angle S is (3x)°.
What is the value of x?

Answers

Answer:

x = 15°

Step-by-step explanation:

(5x-15) + 75 + 3x = 180

8x + 60 = 180

8x = 120

x = 15°

HELP EASY ASAP MULTIPLE CHOICE

Answers

Answer:

It's the first one 18/3 =6

Step-by-step explanation:

This is the only answer that has all integers or whole numbers. Hope this helps!

Answer:

Its the fisrt one cause its the only one with a whole number.

Step-by-step explanation:

How do you find what p is?

Answers

Answer:

p > -3

Step-by-step explanation:

This is an alternating p-series.  It is absolutely convergent if│an│converges.  1 / nᵖ⁺³ converges if the exponent is greater than 1.

p + 3 > 1

p > -2

The series is conditionally convergent if │an│diverges.  1 / nᵖ⁺³ diverges if the exponent is greater than 0 and less than or equal to 1.

0 < p + 3 ≤ 1

-3 < p ≤ -2

Combining these two, we get p > -3.

What is the relationship between x and 86?

Answers

Answer:

C

Step-by-step explanation:

the answer is C because they are on alternate sides and they are on the exterior meaning the outside.

Answer:

alternate exterior angles

Step-by-step explanation:

Consider the hexagon ABCDEF what is the value of X? What is the measure of angle A? what is the measure of angle BCG?

Answers

Answer:

x = 52.5

<A = 152.5°

<BCG = 45°

Step-by-step Explanation:

Sum of interior angles of n-sided polygon = (n - 2)180

Sum of interior angles of hexagon = (6 - 2)180 = 4(180) = 720°

Therefore, sum of all the interior angles in the given hexagon = 720.

Thus:

3x - 5 + 2x + 140 + 135 + 2x + 30 + x = 720

Solve for x. Add like terms.

8x + 300 = 720

Subtract 300 from both sides

8x = 720 - 300

8x = 420

Divide both sides by 8

8x/8 = 420/8

x = 52.5

<A = (3x - 5)

Plug in the value of x

<A = 3(52.5) - 5 = 152.5°

<BCG = 180 - (2x + 30) (angles on a straight line)

Plug in the value of x

<BCG = 180 - (2(52.5) + 30)

<BCG = 180 - (105 + 30)

<BCG = 180 - 135

<BCG = 45°

Show that each conjecture is false by finding a counterexample.


1.For any integer n, n3 > 0.


2. Each angle in a right triangle has a different measure.


3.For many years in the U.S, each bank printed its own currency. The variety of different bills led to widespread counterfeiting. By the time of the Civil War, a significant fraction of the currency in circulation was counterfeit. If one soldier had 48 bills, 16 of which were counterfeit, and another soldier had 39 bills, 13 of which were counterfeit, make a conjecture about what fraction of bills were counterfeit at the time of the Civil War.

Answers

Answer:

1.   [tex]n: n^3 > 0[/tex] is not true for negative integers

2.  [tex]45\ and\ 45[/tex] show that all angles do not have different measure

3.  [tex]Fraction = \frac{1}{3}[/tex]

Step-by-step explanation:

Solving (1):

[tex]n: n^3 > 0[/tex]

Take

[tex]n = -1[/tex]

[tex]-1^3 = -1[/tex]

[tex]n: n^3 > 0[/tex] is not true for negative integers

Solving (2):

The angles of a right angled triangle are:

[tex]45, 45\ and\ 90[/tex]

[tex]45\ and\ 45[/tex] show that all angles do not have different measure

Solving (3):

When

[tex]Bills = 48[/tex]

[tex]Counterfeit = 16[/tex]

The fraction is calculated as thus:

[tex]Fraction = \frac{Counterfeit}{Bills}[/tex]

[tex]Fraction = \frac{16}{48}[/tex]

[tex]Fraction = \frac{1}{3}[/tex]

Similarly, when

[tex]Bills = 39[/tex]

[tex]Counterfeit = 13[/tex]

[tex]Fraction = \frac{Counterfeit}{Bills}[/tex]

[tex]Fraction = \frac{13}{39}[/tex]

[tex]Fraction = \frac{1}{3}[/tex]

whats 62.7×2.45......?

Answers

Answer:

153.615

Step-by-step explanation:

62.7 × 2.45=153.615

ayer 100% A company sells cases of tomato juice. Each case contains 24 cans of juice and sells for $18. The equation y = 18x represents the total price ( y), in dollars, when x number of cases are sold. Choose all statements about the situation that are correct based on the information. A. The cost per can is dependent on the number of cases sold. B. The number of cans sold is dependent on the price per can. C. The total price for a sale is dependent on the number of cases sold. D. The number of cases sold is independent from the number of cans in each case.​

Answers

Answer:

c

Step-by-step explanation:

Answer:

C is the answer

Please help me with all the answers please say like 1: the answer 2: the answer like that thank you so much

Answers

1: 8200,82000,820000 yen
2:12
3:8
4:6
5:4 months

HELP URGENT !!!!!!!!!!!

Answers

The answer is B. -14*1=-14 and -14+1=-13.
I believe the answer is B. Please correct me if I’m wrong! Good luck!

CONSTRUCTED-RESPONSE ITEM 14. Four people each deliver food to people's homes.
19
Constructed Responses
..
Curtis charges a flat fee of $2.50 for each delivery plus $0.20 per mile for each mile he drives. For one delivery, Curtis drives 6 miles.
...
A. How much does Curtis charge to deliver the food?

Answers

Answer:

I'm Pretty Sure it is $16.20

Step-by-step explanation:

First, you add 2.50 + 0.20 and you get 2.70. Then, you get 2.70 x 6 miles and you get 16.20. Hope this helps! :)

Can someone please help me I need help and I can’t understand this plsssss

Answers

1a. Multiply a variable by 13, add 2.

1b. Subtract two from both sides, divide both sides by 13. X=3

2a. Subtract 3 from a variable; divide by 5

2a. Multiply both sides by 5,add 3 to each side. x=-2
Solution (1) :

Solution (a) :

step 1. Multiply the variable x with 13 .

step 2. Add 2 to it .

Solution (b) :

Step 1 : Subtract 2 from both sides : 13x + 2 - 2 = 41 - 2 = 39

Step 2 : Divide both the sides by 13 : 13x ÷ 13 = 39 ÷ 13 = 3

x = 3

Solution (2) :

Solution (a) :

Step 1. Subtract 3 from the variable .

Step 2. Divide the result by 5 .

Solution (b) :

[tex]Step \: 1. \: add\: 3 \: to \: both \: the \: sides : \frac{x - 3}{5} + 3 = - 1 + 3

[/tex]

Step 2 : Multiply both the sides by 5 : 5 × 1/5x = 5 × -2/5

x = -2

11. Factor for

12 – 8.1 + 16​

Answers

I’m going to go ahead and say the answer is 19.9... sorry if it’s wrong.

Answer:

2(6-4.05+8)

Step-by-step explanation:

2(6-48+43.05+8)

Quadrilateral GHJK has vertices G(2, 3), H(8, 2), J(6, 8), and K(3, 6). It is transformed according to the rule T(–4, –5).

What are the coordinates of G”?

(–7, 3)
(–2, 2)
(–1, –7)
(2, –2)

Answers

Answer:

the transformation T(a, b) can be change the pre image to the final image by following the rule as we explain below

if A(x, y) is a pre image, then A ' ( x', y') is its image, and x' = a+x, y' = b + y, where because of T(a, b) transformation

in our case Quadrilateral GHJK has vertices G(2, 3), so the coordinates of G”

can be found with

T(-4, -5) applied to G(2, 3) ⇒ G' (-4 +2, -5+3)=G' (-2, -2)

T(-4, -5) applied to G' (-2, -2) ⇒ G"(-4-2, -5-2) =G"(-6, -7)

the coordinates of G” are (-6, -7)

Step-by-step explanation:

i hope it helps?

Answer:

its c on edg (-1, -7)

Step-by-step explanation:

Other Questions
15 points! :)find the value of x for each figure. each figure is in the centimeters(cm). not to scale.answer this, don't answer if you don't know CAM charges for retail leases in a shopping mall must be calculated. The retail mall consists of a total area of 2.8 million square feet, of which 800,000 square feet has been leased to anchor tenants that have agreed to pay $2 per rentable square foot in CAM charges. In-line tenants occupy 1.3 million square feet, and the remainder is a common area, which the landlord believeswill require $8 per square foot to maintain and operate each year. If the owner is to cover total CAM charges, how much will in-line tenants have to pay per square foot? Plz Need Help Fast, I will mark brainilest 44. Skiing A skier is trying to decide whether or not to buy aseason ski pass. A daily pass costs $67. A season ski passcosts $350. The skier would have to rent skis with either pass for $25 per day. How many dayswould the skier have to go skiing in order to make the season pass less expensive thandaily passes? Find the Unit Rate.216 diving students to 12 instructors Read this excerpt from "Rules of the Game."Ali-ya. So shame be with mother? She grasped my hand even tighter as she glared at me.How does the use of the word glared affect the meaning of the text?A.It emphasizes how angry Waverly's mother is with her daughter.B.It shows how sad Waverly's mother is about Waverly's disrespect.C.It illustrates how frightened Waverly's mother feels about her daughter's response.D.It describes how proud Waverly's mother is with her daughter Helppp asappp pleaseee. What is the slope of a line perpendicular to the line whose equation is2x y = -8. Fully reduce your answer. how to solve this it says solve 6! water evaporates from plants by a process known as how does the relationship between Dorothy and the tinman shape Dorothy? help pls xd- 8x-13+4+1=90 helppppppp Evaluate the expression.6242 6242 How did the US Constitution and the Magna Carta both limit the power of leaders?O Both gave people a voice in government.O Both created separate branches of government.O Both established checks and balances on the people.O Both formed judicial systems ruled by a congress. if y= 2x - 3, find the value of y when x=2 fill in the blanks with the words in parenthesis Today, (no nations, few nations) _________________ have a government that is a direct democracy because it is impractical to bring all citizens together for a meeting due to large populations. Find the missing terms in the sequence:18, -72, 288, Jack refuses to hunt because Ralph asks him to.True orFalse A state makes license plates with five letters followed by one number with no repetitions of the letters permitted. How many different possibilities are there Which of the following is not a correct safety guideline to follow when exercising?OA.Wear protective gear.Do not eat a large meal immediately before exercising. SinOB.O c.Do not drink water while exercising.OD.Follow traffic rules. Where are you most likely to find ligaments in the body?A joints between long bonesB joints between two or more irregular bonesC joints between a long bone and a short boneD joints between muscles and bones